Difference between revisions of "2000 AMC 10 Problems/Problem 11"

(Problem)
(duplicate)
 
(2 intermediate revisions by 2 users not shown)
Line 1: Line 1:
==Problem==
+
#REDIRECT [[2000 AMC 12 Problems/Problem 6]]
 
 
Two different prime numbers between <math>4</math> and <math>18</math> are chosen. When their sum is subtracted from their product, which of the following numbers could be obtained?
 
 
 
<math>\mathrm{(A)}\ 21 \qquad\mathrm{(B)}\ 60 \qquad\mathrm{(C)}\ 119 \qquad\mathrm{(D)}\ 180 \qquad\mathrm{(E)}\ 231</math>
 
 
 
==Solution==
 
 
 
Two prime numbers between <math>4</math> and <math>18</math> are both odd.
 
 
 
odd*odd=odd.
 
 
 
odd-odd-odd=odd.
 
 
 
Thus, we can discard the even choices.
 
 
 
<math>ab-a-b=(a-1)(b-1)-1</math>.
 
 
 
<math>(a-1)(b-1)-1=21,119,231</math>.
 
 
 
Both of these factors are even, so the number +1 must be a multiple of <math>4</math>.
 
 
 
<math>120</math> is the only possiblity.
 
 
 
<math>11,13</math> satisfy this, <math>11*13-11-13=143-24=119</math>.
 
 
 
C
 
 
 
==See Also==
 
 
 
{{AMC10 box|year=2000|num-b=10|num-a=12}}
 

Latest revision as of 23:42, 26 November 2011